Graphing inequality with mod function

Click For Summary
To graph the inequality |y| + 1/2 >= e - |x|, it can be rewritten as y >= e - |x| - 1/2 or y <= -e + |x| + 1/2. The function is symmetric across all quadrants and intersects the axes at +ln2 and -ln2 on the x-axis, and 1/2 and -1/2 on the y-axis. The complexity arises from the mixed graphs, making it challenging to identify the correct region. Considering the intervals for the different inequalities is crucial for accurate graphing. Understanding these transformations aids in visualizing the solution set effectively.
aim1732
Messages
428
Reaction score
2
How can we graph this inequality - |y|+1/2>=e-|x| ?
I drew the function(actually a combination of functions) for equality. It would be symmetric in all quadrants and intersect the axes at +ln2 and -ln2(x-axis) and 1/2 and -1/2(y-axis).However since the various graphs are mixed up it is hard pinpoint what region to take.

I have attached one half of the actual graph.
The other half is it's reflection on the x-axis.
 

Attachments

  • graph.php.png
    graph.php.png
    1.8 KB · Views: 477
Last edited:
Physics news on Phys.org
aim1732 said:
How can we graph this inequality - |y|+1/2>=e-|x| ?
I drew the function(actually a combination of functions) for equality. It would be symmetric in all quadrants and intersect the axes at +ln2 and -ln2(x-axis) and 1/2 and -1/2(y-axis).However since the various graphs are mixed up it is hard pinpoint what region to take.

I have attached one half of the actual graph.
The other half is it's reflection on the x-axis.
Your inequality is equivalent to |y| >= e-|x| - 1/2. This can be rewritten as
y >= e-|x| - 1/2 or -y >= e-|x| - 1/2

So y >= e-|x| - 1/2 or y <= -e-|x| + 1/2

Does that help?
 
Yes I should have considered the intervals for the different inequalities.Thanks a lot.
 
Question: A clock's minute hand has length 4 and its hour hand has length 3. What is the distance between the tips at the moment when it is increasing most rapidly?(Putnam Exam Question) Answer: Making assumption that both the hands moves at constant angular velocities, the answer is ## \sqrt{7} .## But don't you think this assumption is somewhat doubtful and wrong?

Similar threads

Replies
3
Views
1K
  • · Replies 5 ·
Replies
5
Views
1K
  • · Replies 11 ·
Replies
11
Views
2K
  • · Replies 4 ·
Replies
4
Views
1K
  • · Replies 9 ·
Replies
9
Views
2K
  • · Replies 4 ·
Replies
4
Views
2K
  • · Replies 19 ·
Replies
19
Views
2K
  • · Replies 6 ·
Replies
6
Views
2K
  • · Replies 15 ·
Replies
15
Views
2K
  • · Replies 3 ·
Replies
3
Views
2K